Exam 3 Prep U Flashcards

1
Q

A nurse is preparing a presentation for a local community group about hepatitis. Which of the following would the nurse include?

A

Hepatitis C increases a person’s risk for liver cancer.
Infection with hepatitis C increases the risk of a person developing hepatic (liver) cancer. Hepatitis A is transmitted primarily by the oral–fecal route; hepatitis B is frequently spread by sexual contact and infected blood. Hepatitis E is similar to hepatitis A whereas hepatitis G is similar to hepatitis C.

How well did you know this?
1
Not at all
2
3
4
5
Perfectly
2
Q

Which is an age-related change of the hepatobiliary system?

A

Decreased blood flow
Explanation:
Age-related changes of the hepatobiliary system include decreased blood flow, decreased drug clearance capability, increased presence of gall stones, and a steady decrease in the size and weight of the liver.

How well did you know this?
1
Not at all
2
3
4
5
Perfectly
3
Q

A nurse educator is providing an in-service to a group of nurses working on a medical floor that specializes in liver disorders. What is an important education topic regarding ingestion of medications?

A

metabolism of medications
Explanation:
Careful evaluation of the client’s response to drug therapy is important because the malfunctioning liver cannot metabolize many substances.

How well did you know this?
1
Not at all
2
3
4
5
Perfectly
4
Q

A client with cirrhosis has portal hypertension, which is causing esophageal varices. What is the goal of the interventions that the nurse will provide?

A

Reduce fluid accumulation and venous pressure.
Explanation:
Methods of treating portal hypertension aim to reduce fluid accumulation and venous pressure. There is no cure for cirrhosis; treating the esophageal varices is only a small portion of the overall objective. Promoting optimal neurologic function will not reduce portal hypertension.

How well did you know this?
1
Not at all
2
3
4
5
Perfectly
5
Q

In what location would the nurse palpate for the liver?

A

Right upper quadrant
Explanation:
The liver may be palpable in the right upper quadrant. A palpable liver presents as a firm, sharp ridge with a smooth surface.

How well did you know this?
1
Not at all
2
3
4
5
Perfectly
6
Q

When performing a physical examination on a client with cirrhosis, a nurse notices that the client’s abdomen is enlarged. Which of the following interventions should the nurse consider?

A

Measure abdominal girth according to a set routine.
Explanation:
If the abdomen appears enlarged, the nurse measures it according to a set routine. The nurse reports any change in mental status or signs of gastrointestinal bleeding immediately. It is not essential for the client to take laxatives unless prescribed. The client’s food intake does not affect the size of the abdomen in case of cirrhosis.

How well did you know this?
1
Not at all
2
3
4
5
Perfectly
7
Q

An important message for any nurse to communicate is that drug-induced hepatitis is a major cause of acute liver failure. The medication that is the leading cause is:

A

Acetaminophen
Explanation:
Although any medication can affect liver function, use of acetaminophen (found in many over-the-counter medications used to treat fever and pain) has been identified as the leading cause of acute liver failure. Other medications commonly associated with liver injury include anesthetic agents, medications used to treat rheumatic and musculoskeletal disease, antidepressants, psychotropic medications, anticonvulsants, and antituberculosis agents.

How well did you know this?
1
Not at all
2
3
4
5
Perfectly
8
Q

Clients with chronic liver dysfunction have problems with insufficient vitamin intake. Which may occur as a result of vitamin C deficiency?

A

Scurvy
Explanation:
Scurvy may result from a vitamin C deficiency. Night blindness, hypoprothrombinemia, and beriberi do not result from a vitamin C deficiency.

How well did you know this?
1
Not at all
2
3
4
5
Perfectly
9
Q

A client with right upper quadrant pain and weight loss is diagnosed with liver cancer. For which treatment will the nurse prepare the client when it is determined that the disease is confined to one lobe of the liver?

A

Liver resection

Explanation:
Surgical resection is the treatment of choice when liver cancer is confined to one lobe of the liver and the function of the remaining liver is considered adequate for postoperative recovery. The use of external-beam radiation for the treatment of liver tumors has been limited by the radiosensitivity of normal hepatocytes and the risk of destruction of normal liver parenchyma. Studies of clients with advanced cases of liver cancer have shown that the use of systemic chemotherapeutic agents leads to poor outcomes. Laser hyperthermia has been used to treat hepatic metastases.

How well did you know this?
1
Not at all
2
3
4
5
Perfectly
10
Q

The nurse identifies which type of jaundice in an adult experiencing a transfusion reaction?

A

Hemolytic
Explanation:
Hemolytic jaundice occurs because, although the liver is functioning normally, it cannot excrete the bilirubin as quickly as it is formed. This type of jaundice is encountered in clients with hemolytic transfusion reactions and other hemolytic disorders. Obstructive and hepatocellular jaundice are the result of liver disease. Nonobstructive jaundice occurs with hepatitis.

How well did you know this?
1
Not at all
2
3
4
5
Perfectly
11
Q

The nurse is administering medications to a client that has elevated ammonia due to cirrhosis of the liver. What medication will the nurse give to detoxify ammonium and to act as an osmotic agent?

A

Lactulose
Explanation:
Lactulose is administered to detoxify ammonium and to act as an osmotic agent, drawing water into the bowel, which causes diarrhea in some clients. Potassium-sparing diuretics such as spironolactone are used to treat ascites. Cholestyramine is a bile acid sequestrant and reduces pruritus. Kanamycin decreases intestinal bacteria and decreases ammonia but does not act as an osmotic agent.

How well did you know this?
1
Not at all
2
3
4
5
Perfectly
12
Q

The nurse is caring for a patient with ascites due to cirrhosis of the liver. What position does the nurse understand will activate the renin-angiotensin aldosterone and sympathetic nervous system and decrease responsiveness to diuretic therapy?

A

Upright
Explanation:
In patients with ascites, an upright posture is associated with activation of the renin–angiotensin–aldosterone system and sympathetic nervous system (Porth & Matfin, 2009). This causes reduced renal glomerular filtration and sodium excretion and a decreased response to loop diuretics.

How well did you know this?
1
Not at all
2
3
4
5
Perfectly
13
Q

Which is the most common cause of esophageal varices?

A

Portal hypertension
Explanation:
Esophageal varices are almost always caused by portal hypertension, which results from obstruction of the portal circulation within the damaged liver. Jaundice occurs when the bilirubin concentration in the blood is abnormally elevated. Ascites results from circulatory changes within the diseased liver. Asterixis is an involuntary flapping movement of the hands associated with metabolic liver dysfunction.

How well did you know this?
1
Not at all
2
3
4
5
Perfectly
14
Q

A client reporting shortness of breath is admitted with a diagnosis of cirrhosis. A nursing assessment reveals an enlarged abdomen with striae, an umbilical hernia, and 4+ pitting edema of the feet and legs. What is the most important data for the nurse to monitor?

A

Albumin
Explanation:
With the movement of albumin from the serum to the peritoneal cavity, the osmotic pressure of the serum decreases. This, combined with increased portal pressure, results in movement of fluid into the peritoneal cavity. The low oncotic pressure caused by hypoalbuminemia is a major pathophysiologic factor in the development of ascites and edema.

How well did you know this?
1
Not at all
2
3
4
5
Perfectly
15
Q

The nurse is caring for a patient with cirrhosis of the liver and observes that the patient is having hand-flapping tremors. What does the nurse document this finding as?

A

Asterixis
Explanation:
Asterixis, an involuntary flapping of the hands, may be seen in stage II encephalopathy (Fig. 49-13).

How well did you know this?
1
Not at all
2
3
4
5
Perfectly
16
Q

Ammonia, the major etiologic factor in the development of encephalopathy, inhibits neurotransmission. Increased levels of ammonia are damaging to the body. The largest source of ammonia is from:

A

The digestion of dietary and blood proteins.
Explanation:
Circumstances that increase serum ammonia levels tend to aggravate or precipitate hepatic encephalopathy. The largest source of ammonia is the enzymatic and bacterial digestion of dietary and blood proteins in the GI tract. Ammonia from these sources increases as a result of GI bleeding (i.e., bleeding esophageal varices, chronic GI bleeding), a high-protein diet, bacterial infection, or uremia.

How well did you know this?
1
Not at all
2
3
4
5
Perfectly
17
Q

A client with viral hepatitis A is being treated in an acute care facility. Because the client requires enteric precautions, the nurse should:

A

wash her hands after touching the client.
Explanation:
To maintain enteric precautions, the nurse must wash her hands after touching the client or potentially contaminated articles and before caring for another client. A private room is warranted only if the client has poor hygiene — for instance, if the client is unlikely to wash the hands after touching infective material or is likely to share contaminated articles with other clients. For enteric precautions, the nurse need not wear a mask and must wear a gown only if soiling from fecal matter is likely.

How well did you know this?
1
Not at all
2
3
4
5
Perfectly
18
Q

Which of the following would the nurse expect to assess in a conscious client with hepatic encephalopathy?

A

Asterixis
Explanation:
Hepatic encephalopathy is manifested by numerous central nervous system effects including disorientation, confusion, mood swings, reversed day–night sleep patterns with sleep occurring during the day, agitation, memory loss, a flapping tremor called asterixis, a positive Babinski reflex, sulfurous breath odor (referred to as fetor hepaticus), and lethargy. As hepatic encephalopathy becomes more severe, the client becomes stuporous and eventually comatose.

How well did you know this?
1
Not at all
2
3
4
5
Perfectly
19
Q

A client is being prepared to undergo laboratory and diagnostic testing to confirm the diagnosis of cirrhosis. Which test would the nurse expect to be used to provide definitive confirmation of the disorder?

A

Liver biopsy
Explanation:
A liver biopsy which reveals hepatic fibrosis is the most conclusive diagnostic procedure. Coagulation studies provide information about liver function but do not definitively confirm the diagnosis of cirrhosis. Magnetic resonance imaging and radioisotope liver scan help to support the diagnosis but do not confirm it. These tests provide information about the liver’s enlarged size, nodular configuration, and distorted blood flow.

How well did you know this?
1
Not at all
2
3
4
5
Perfectly
20
Q

Which type of jaundice seen in adults is the result of increased destruction of red blood cells?

A

Hemolytic
Explanation:
Hemolytic jaundice results because, although the liver is functioning normally, it cannot excrete the bilirubin as quickly as it is formed. Obstructive and hepatocellular jaundice are results of liver disease. Nonobstructive jaundice occurs with hepatitis.

How well did you know this?
1
Not at all
2
3
4
5
Perfectly
21
Q

In actively bleeding patients with esophageal varices, the initial drug of therapy is usually:

A

Sandostatin
Explanation:
In an actively bleeding patient, medications are given initially because they can be obtained and given more quickly than other therapies. Sandostatin, a synthetic analog of the hormone somatostatin, is effective in decreasing bleeding from esophageal varices and lacks the vasoconstrictive effects of vasopressin. Because of this safety and efficacy profile, octreotide is considered the preferred treatment regimen for immediate control of variceal bleeding.

How well did you know this?
1
Not at all
2
3
4
5
Perfectly
22
Q

A client with carcinoma of the head of the pancreas is scheduled for surgery. Which of the following should a nurse administer to the client before surgery?

A

Vitamin K
Explanation:
Clients with carcinoma of the head of the pancreas typically require vitamin K before surgery to correct a prothrombin deficiency. Potassium would be given only if the client’s serum potassium levels were low. Oral bile acids are not prescribed for a client with carcinoma of the head of the pancreas; they are given to dissolve gallstones. Vitamin B has no implications in the surgery.

How well did you know this?
1
Not at all
2
3
4
5
Perfectly
23
Q

A critically ill client is diagnosed with acute liver failure caused by an overdose of acetaminophen. Which treatment will the nurse anticipate being prescribed for the client?

A

N-acetylcysteine

Explanation:
Acute hepatic failure or acute liver failure (ALF) is the clinical syndrome of sudden and severely impaired liver function in a person who was previously healthy. Supporting the client in the ICU and assessing the indications for and feasibility of liver transplantation are hallmarks of management. The use of antidotes for certain conditions may be indicated, such as N-acetylcysteine for acetaminophen toxicity. Penicillin is used for mushroom poisoning. Prostaglandins are used to enhance hepatic blood flow. Plasma exchange is used to correct coagulopathy, reduce serum ammonia levels, and stabilize the client awaiting liver transplantation.

How well did you know this?
1
Not at all
2
3
4
5
Perfectly
24
Q

A client has just been diagnosed with hepatitis A. On assessment, the nurse expects to note:

A

anorexia, nausea, and vomiting.
Explanation:
Early hallmark signs and symptoms of hepatitis A include anorexia, nausea, vomiting, fatigue, and weakness. Abdominal pain may occur but doesn’t radiate to the shoulder. Eructation and constipation are common in gallbladder disease, not hepatitis A. Abdominal ascites is a sign of advanced hepatic disease, not an early sign of hepatitis A.

How well did you know this?
1
Not at all
2
3
4
5
Perfectly
25
Q

Which of the following is the most effective strategy to prevent hepatitis B infection?

A

Vaccine
Explanation:
The most effective strategy to prevent hepatitis B infection is through vaccination. Recommendations to prevent transmission of hepatitis B include vaccination of sexual contacts of individuals with chronic hepatitis, use of barrier protection during sexual intercourse, avoidance of sharing toothbrushes, razors with others, and covering open sores or skin lesions.

How well did you know this?
1
Not at all
2
3
4
5
Perfectly
26
Q

The mode of transmission of hepatitis A virus (HAV) includes which of the following?

A

Fecal-oral
Explanation:
The mode of transmission of hepatitis A virus (HAV) occurs through fecal-oral route, primarily through person to person contact and/or ingestion of fecal contaminated food or water. Hepatitis B virus (HBV) is transmitted primarily through blood. HBV can be found in blood, saliva, semen, and can be transmitted through mucous membranes and breaks in the skin.

How well did you know this?
1
Not at all
2
3
4
5
Perfectly
27
Q

What initial measure can the nurse implement to reduce risk of injury for a client with liver disease?

A

Pad the side rails on the bed
Explanation:
Padding the side rails can reduce injury if the client becomes agitated or restless. Restraints would not be an initial measure to implement. Four side rails are considered a restraint, and this would not be an initial measure to implement. Family and friends generally assist in calming a client.

How well did you know this?
1
Not at all
2
3
4
5
Perfectly
28
Q

What is the recommended dietary treatment for a client with chronic cholecystitis?

A

low-fat diet
Explanation:
The bile secreted from the gallbladder helps the body absorb and break down dietary fats. If the gallbladder is not functioning properly, then it will not secrete enough bile to help digest the dietary fat. This can lead to further complications; therefore, a diet low in fat can be used to prevent complications.

How well did you know this?
1
Not at all
2
3
4
5
Perfectly
29
Q

A patient with severe chronic liver dysfunction comes to the clinic with bleeding of the gums and blood in the stool. What vitamin deficiency does the nurse suspect the patient may be experiencing?

A

Vitamin K deficiency
Explanation:
Vitamin A deficiency results in night blindness and eye and skin changes. Thiamine deficiency leads to beriberi, polyneuritis, and Wernicke-Korsakoff psychosis. Riboflavin deficiency results in characteristic skin and mucous membrane lesions. Pyridoxine deficiency results in skin and mucous membrane lesions and neurologic changes. Vitamin C deficiency results in the hemorrhagic lesions of scurvy. Vitamin K deficiency results in hypoprothrombinemia, characterized by spontaneous bleeding and ecchymoses. Folic acid deficiency results in macrocytic anemia.

How well did you know this?
1
Not at all
2
3
4
5
Perfectly
30
Q

A client with cirrhosis has a massive hemorrhage from esophageal varices. Balloon tamponade is used temporarily to control hemorrhage and stabilize the client. In planning care, the nurse gives the highest priority to which goal?

A

Maintaining the airway
Explanation:
Esophageal varices are almost always caused by portal hypertension, which results from obstruction of the portal circulation within the damaged liver. Maintaining the airway is the highest priority because oxygenation is essential for life. The airway can be compromised by possible displacement of the tube and the inflated balloon into the oropharynx, which can cause life-threatening obstruction of the airway and asphyxiation.

How well did you know this?
1
Not at all
2
3
4
5
Perfectly
31
Q

The nurse is caring for a client with chronic pancreatitis. Which symptom would indicate the client has developed secondary diabetes?

A

Increased appetite and thirst
Explanation:
When secondary diabetes develops in a client with chronic pancreatitis, the client experiences increased appetite, thirst, and urination. Vomiting, diarrhea, low blood pressure and pulse, and constipation do not indicate the development of secondary diabetes.

How well did you know this?
1
Not at all
2
3
4
5
Perfectly
32
Q

A client is seeing the physician for a suspected tumor of the liver. What laboratory study results would indicate that the client may have a primary malignant liver tumor?

A

Elevated alpha-fetoprotein
Explanation:
Alpha-fetoprotein, a serum protein normally produced during fetal development, is a marker that, if elevated, can induce a primary malignant liver tumor. Total bilirubin and serum enzyme levels may be elevated. White blood cell count elevation would indicate an inflammatory response.

How well did you know this?
1
Not at all
2
3
4
5
Perfectly
33
Q

A client with acute liver failure exhibits confusion, a declining level of consciousness, and slowed respirations. The nurse finds him very difficult to arouse. The diagnostic information which best explains the client’s behavior is:

A

subnormal serum glucose and elevated serum ammonia levels.
Explanation:
In acute liver failure, serum ammonia levels increase because the liver can’t adequately detoxify the ammonia produced in the GI tract. In addition, serum glucose levels decline because the liver isn’t capable of releasing stored glucose. Elevated serum ammonia and subnormal serum glucose levels depress the level of a client’s consciousness. Elevated liver enzymes, low serum protein level, subnormal clotting factors and platelet count, elevated blood urea nitrogen and creatine levels, and hyperglycemia aren’t as directly related to the client’s level of consciousness.

How well did you know this?
1
Not at all
2
3
4
5
Perfectly
34
Q

A young client with anorexia, fatigue, and jaundice is diagnosed with hepatitis B and has just been admitted to the hospital. The client asks the nurse how long the stay in the hospital will be. In planning care for the client, the nurse identifies impaired psychosocial issues and assigns the highest priority to which client outcome?

A

Minimizing social isolation
Explanation:
The nurse identifies psychosocial issues and concerns, particularly the effects of separation from family and friends if the client is hospitalized during the acute and infective stages. Convalescence may be prolonged, with complete symptomatic recovery sometimes requiring 3 to 4 months or longer. Even if not hospitalized, the client will be unable to attend school and/or work and must avoid sexual contact. Planning is required to minimize social isolation.

How well did you know this?
1
Not at all
2
3
4
5
Perfectly
35
Q

A client who has worked for a company that produces paint and varnishing compounds for 24 years is visiting the clinic reporting chronic fatigue, dyspepsia, diarrhea, and a recently developing yellowing of the skin and sclera. The client reports clay-colored stools and frequent nosebleeds. Which type of cirrhosis is the likely cause of the client’s symptoms?

A

postnecrotic
Explanation:
Postnecrotic cirrhosis results from destruction of liver cells secondary to infection (e.g., hepatitis), metabolic liver disease, or exposure to hepatotoxins or industrial chemicals. Alcoholic cirrhosis develops as a consequence of long-term alcohol use disorder. Respiratory cirrhosis is not a type of cirrhosis. Biliary cirrhosis is less common than other types and is associated with scarring in the bile ducts.

How well did you know this?
1
Not at all
2
3
4
5
Perfectly
36
Q

The nurse is assessing a client with cirrhosis of the liver. Which stool characteristic would the nurse expect the client to report?

A

Clay-colored or whitish
Explanation:
Many clients report passing clay-colored or whitish stools as a result of no bile in the gastrointestinal tract. The other stool colors would not be indicators of obstructive jaundice but may indicate other GI tract disorders.

How well did you know this?
1
Not at all
2
3
4
5
Perfectly
37
Q

When caring for a client with advanced cirrhosis and hepatic encephalopathy, which assessment finding should the nurse report immediately?

A

Change in the client’s handwriting and/or cognitive performance
Explanation:
The earliest symptoms of hepatic encephalopathy include mental status changes and motor disturbances. The client will appear confused and unkempt and have altered mood and sleep patterns. Neurologic status should be assessed frequently. Mental status is monitored by the nurse keeping the client’s daily record of handwriting and arithmetic performance. The nurse should report any change in mental status immediately. Chronic fatigue, anorexia, dyspepsia, nausea, vomiting, and diarrhea or constipation with accompanying weight loss are regular symptoms of cirrhosis.

How well did you know this?
1
Not at all
2
3
4
5
Perfectly
38
Q

A nurse is caring for a client with cirrhosis. The nurse assesses the client at noon and discovers that the client is difficult to arouse and has an elevated serum ammonia level. The nurse should suspect which situation?

A

The client’s hepatic function is decreasing.
Explanation:
The decreased level of consciousness caused by an increased serum ammonia level indicates hepatic disfunction. If the client didn’t take his morning dose of lactulose, he wouldn’t have elevated ammonia levels and decreased level of consciousness this soon. These assessment findings don’t indicate that the client is relaxed or avoiding the nurse.

How well did you know this?
1
Not at all
2
3
4
5
Perfectly
39
Q

A nurse is caring for a client with cholelithiasis. Which sign indicates obstructive jaundice?

A

Clay-colored stools
Explanation:
Obstructive jaundice develops when a stone obstructs the flow of bile in the common bile duct. When the flow of bile to the duodenum is blocked, the lack of bile pigments results in a clay-colored stool. In obstructive jaundice, urine tends to be dark amber (not straw-colored) as a result of soluble bilirubin in the urine. Hematocrit levels aren’t affected by obstructive jaundice. Because obstructive jaundice prevents bilirubin from reaching the intestine (where it’s converted to urobilinogen), the urine contains no urobilinogen.

How well did you know this?
1
Not at all
2
3
4
5
Perfectly
40
Q

Which medication is used to decrease portal pressure, halting bleeding of esophageal varices?

A

Vasopressin
Explanation:
Vasopressin may be the initial therapy for esophageal varices because it produces constriction of the splanchnic arterial bed and decreases portal hypertension. Nitroglycerin has been used to prevent the side effects of vasopressin. Spironolactone and cimetidine do not decrease portal hypertension.

How well did you know this?
1
Not at all
2
3
4
5
Perfectly
41
Q

The nurse is caring for a client with cirrhosis. Which assessment findings indicate that the client has deficient vitamin K absorption caused by this hepatic disease?

A

Purpura and petechiae
Explanation:
A hepatic disorder, such as cirrhosis, may disrupt the liver’s normal use of vitamin K to produce prothrombin (a clotting factor). Consequently, the nurse should monitor the client for signs of bleeding, including purpura and petechiae. Dyspnea and fatigue suggest anemia. Ascites and orthopnea are unrelated to vitamin K absorption. Gynecomastia and testicular atrophy result from decreased estrogen metabolism by the diseased liver.

How well did you know this?
1
Not at all
2
3
4
5
Perfectly
42
Q

A client with liver cirrhosis develops ascites. Which medication will the nurse prepare teaching for this client?

A

Spironolactone

Explanation:
The use of diuretic agents along with sodium restriction is successful in 90% of clients with ascites. Spironolactone, an aldosterone-blocking agent, is most often the first-line therapy in clients with ascites from cirrhosis. When used with other diuretic agents, spironolactone helps prevent potassium loss. Oral diuretic agents such as furosemide may be added but should be used cautiously because long-term use may induce severe hyponatremia (sodium depletion). Acetazolamide and ammonium chloride are contraindicated because of the possibility of precipitating hepatic encephalopathy and coma.

How well did you know this?
1
Not at all
2
3
4
5
Perfectly
43
Q

A nurse cares for clients with hematological disorders and notes that women are diagnosed with hemochromatosis at a much lower rate than men. What is the primary reason for this?

A

Women lose iron through menstrual cycles
Explanation:
Hemochromatosis is a genetic condition where excess iron is absorbed in the GI tract and deposited in various organs, making them dysfunctional. Women are often less affected than men because women lose excess iron through their menstrual cycles. The other answer choices are not correct reasons why women are impacted less than men with hemochromatosis.

How well did you know this?
1
Not at all
2
3
4
5
Perfectly
44
Q

Which of the following describes a red blood cell (RBC) that has pale or lighter cellular contents?

A

Hypochromic
Explanation:
An RBC that has pale or lighter cellular contents is hypochromic. A normocytic RBC is normal or average in size. A microcytic RBC is smaller than normal. Hyperchromic is used to describe an RBC that has darker cellular contents.

How well did you know this?
1
Not at all
2
3
4
5
Perfectly
45
Q

The nurse is screening donors for blood donation. Which client is an acceptable donor for blood?

A

Reports having a cold 1 month ago that resolved quickly
Explanation:
Donors must meet certain requirements to be able to donate blood. A client should be in good health, such as the client who had a cold more than 1 month ago that resolved quickly. Those excluded from donating blood have a history of viral hepatitis, report a blood transfusion within 12 months, and had a dental extraction within 72 hours. The reason for exclusion is that they are at increased risk of transmitting an infectious disease.

How well did you know this?
1
Not at all
2
3
4
5
Perfectly
46
Q

A patient with end-stage kidney disease (ESKD) has developed anemia. What laboratory finding does the nurse understand to be significant in this stage of anemia?

A

Creatinine level of 6 mg/100 mL
Explanation:
The degree of anemia in patients with end-stage renal disease varies greatly; however, in general, patients do not become significantly anemic until the serum creatinine level exceeds 3 mg/100 mL.

How well did you know this?
1
Not at all
2
3
4
5
Perfectly
47
Q

A nurse is caring for a client admitted with pernicious anemia. Which set of findings should the nurse expect when assessing the client?

A

Pallor, tachycardia, and a sore tongue
Explanation:
Pallor, tachycardia, and a sore tongue are all characteristic findings in pernicious anemia. Other clinical manifestations include anorexia; weight loss; a smooth, beefy red tongue; a wide pulse pressure; palpitations; angina pectoris; weakness; fatigue; and paresthesia of the hands and feet. Bradycardia, reduced pulse pressure, weight gain, and double vision aren’t characteristic findings in pernicious anemia.

How well did you know this?
1
Not at all
2
3
4
5
Perfectly
48
Q

A client with megaloblastic anemia reports mouth and tongue soreness. What instruction will the nurse give the client regarding eating while managing the client’s symptoms?

A

“Eat small amounts of bland, soft foods frequently.”
Explanation:
Because the client with megaloblastic anemia often reports mouth and tongue soreness, the nurse should instruct the client to eat small amounts of bland, soft foods frequently. The other answer choices do not factor in the client’s mouth soreness or need for nutrition.

How well did you know this?
1
Not at all
2
3
4
5
Perfectly
49
Q

A complete blood count is commonly performed before a client goes into surgery. What does this test seek to identify?

A

Abnormally low hematocrit (HCT) and hemoglobin (Hb) levels
Explanation:
Low preoperative HCT and Hb levels indicate the client may require a blood transfusion before surgery. If the HCT and Hb levels decrease during surgery because of blood loss, the potential need for a transfusion increases. Possible renal failure is indicated by elevated BUN or creatinine levels. Urine constituents aren’t found in the blood. Coagulation is determined by the presence of appropriate clotting factors, not electrolytes.

How well did you know this?
1
Not at all
2
3
4
5
Perfectly
50
Q

A nursing instructor is evaluating a student caring for a neutropenic client. The instructor concludes that the nursing student demonstrates accurate knowledge of neutropenia based on which intervention?

A

Monitoring the client’s temperature and reviewing the client’s complete blood count (CBC) with differential
Explanation:
Clients with neutropenia often do not exhibit classic signs of infection. Fever is the most common indicator of infection, yet it is not always present. No definite symptoms of neutropenia appear until the client develops an infection. A routine CBC with differential can reveal neutropenia before the onset of infection.

How well did you know this?
1
Not at all
2
3
4
5
Perfectly
51
Q

The nurse is collecting data for a client who has been diagnosed with iron-deficiency anemia. What subjective findings does the nurse recognize as symptoms related to this type of anemia?

A

“I have difficulty breathing when walking 30 feet.”
Explanation:
Most clients with iron-deficiency anemia have reduced energy, feel cold all the time, and experience fatigue and dyspnea with minor physical exertion. The heart rate usually is rapid even at rest. The CBC and hemoglobin, hematocrit, and serum iron levels are decreased. The client would feel cold and not hot. The client is fatigued and able to sleep often with a decrease in appetite, not an increase.

How well did you know this?
1
Not at all
2
3
4
5
Perfectly
52
Q

A home care nurse visits a client diagnosed with atrial fibrillation who is ordered warfarin. The nurse teaches the client about warfarin therapy. Which statement by the client indicates the need for further teaching?

A

“I’ll eat four servings of fresh, dark green vegetables every day.”
Explanation:
The client requires additional teaching if he states that he’ll eat four servings of dark green vegetables every day. Dark, green vegetables contain vitamin K, which reverses the effects of warfarin. The client should limit his intake to one to two servings per day. The client should report bleeding gums and severe or unexplained bruising, which may indicate an excessive dose of warfarin. The client should use an electric razor to prevent cutting himself while shaving.

How well did you know this?
1
Not at all
2
3
4
5
Perfectly
53
Q

The nurse is instructing a client about taking a liquid iron preparation for the treatment of iron-deficiency anemia. What should the nurse include in the instructions?

A

Dilute the liquid preparation with another liquid such as juice and drink with a straw.
Explanation:
Dilute liquid preparations of iron with another liquid such as juice and drink with a straw to avoid staining the teeth. Avoid taking iron simultaneously with an antacid, which interferes with iron absorption. Drink orange juice or take other forms of vitamin C with iron to promote its absorption. Expect iron to color stool dark green or black.

How well did you know this?
1
Not at all
2
3
4
5
Perfectly
54
Q

A nurse is caring for a client with severe anemia. The client is tachycardic and reports dizziness and exertional dyspnea. What signs and symptoms might develop if this client goes into heart failure?

A

Peripheral edema
Explanation:
Cardiac status should be carefully assessed in clients with anemia. When the hemoglobin level is low, the heart attempts to compensate by pumping faster and harder in an effort to deliver more blood to hypoxic tissue. This increased cardiac workload can result in such symptoms such as tachycardia, palpitations, dyspnea, dizziness, orthopnea, and exertional dyspnea. Heart failure may eventually develop, as evidenced by an enlarged heart (cardiomegaly) and liver (hepatomegaly), and by peripheral edema. Nausea, migraine, and fever are not associated with heart failure.`

How well did you know this?
1
Not at all
2
3
4
5
Perfectly
55
Q

While monitoring a client for the development of disseminated intravascular coagulation (DIC), the nurse should take note of which assessment parameters?

A

Platelet count, prothrombin time, and partial thromboplastin time
Explanation:
The diagnosis of DIC is based on the results of laboratory studies of prothrombin time, platelet count, thrombin time, partial thromboplastin time, and fibrinogen level as well as client history and other assessment factors. Blood glucose levels, WBC count, calcium levels, and potassium levels aren’t used to confirm a diagnosis of DIC.

How well did you know this?
1
Not at all
2
3
4
5
Perfectly
56
Q

A client with pernicious anemia is receiving parenteral vitamin B12 therapy. Which client statement indicates effective teaching about this therapy?

A

“I will receive parenteral vitamin B12 therapy for the rest of my life.”
Explanation:
Because a client with pernicious anemia lacks intrinsic factor, oral vitamin B12 can’t be absorbed. Therefore, parenteral vitamin B12 therapy is recommended and required for life.

How well did you know this?
1
Not at all
2
3
4
5
Perfectly
57
Q

A client admitted to the hospital with abdominal pain, anemia, and bloody stools reports feeling weak and dizzy. The client has rectal pressure and needs to urinate and move their bowels. The nurse should help them:

A

onto the bedpan.
Explanation:
A client who’s dizzy and anemic is at risk for injury because of his weakened state. Assisting him with the bedpan would best meet his needs at this time without risking his safety. The client may fall if walking to the bathroom, left alone to urinate, or trying to stand up.

How well did you know this?
1
Not at all
2
3
4
5
Perfectly
58
Q

The nurse observes a co-worker who always seems to be eating a cup of ice. The nurse encourages the co-worker to have an examination and diagnostic workup with the health care provider. What type of anemia is the nurse concerned the co-worker may have?

A

Iron deficiency anemia
Explanation:
People with iron deficiency anemia may crave ice, starch, or dirt; this craving is known as pica.

How well did you know this?
1
Not at all
2
3
4
5
Perfectly
59
Q

A few minutes after beginning a blood transfusion, a nurse notes that a client has chills, dyspnea, and urticaria. The nurse reports this to the health care provider immediately because the client probably is experiencing which problem?

A

A hemolytic allergic reaction caused by an antigen reaction
Explanation:
Hemolytic allergic reactions are fairly common and may cause chills, fever, urticaria, tachycardia, dyspnea, chest pain, hypotension, and other signs of anaphylaxis a few minutes after blood transfusion begins. Although rare, a hemolytic reaction to mismatched blood can occur, triggering a more severe reaction and, possibly, leading to disseminated intravascular coagulation. A hemolytic reaction to Rh-incompatible blood is less severe and occurs several days to 2 weeks after the transfusion. Bacterial contamination of donor blood causes a high fever, nausea, vomiting, diarrhea, abdominal cramps and, possibly, shock.

How well did you know this?
1
Not at all
2
3
4
5
Perfectly
60
Q

A client reports feeling tired, cold, and short of breath at times. Assessment reveals tachycardia and reduced energy. What would the nurse expect the physician to order?

A

CBC
Explanation:
Most clients with iron-deficiency anemia have reduced energy, feel cold all the time, and experience fatigue and dyspnea with minor physical exertion. The heart rate usually is rapid even at rest. The CBC and hemoglobin, hematocrit, and serum iron levels are decreased. A CBC would be ordered.

How well did you know this?
1
Not at all
2
3
4
5
Perfectly
61
Q

During the review of morning laboratory values for a client reporting severe fatigue and a red, swollen tongue, the nurse suspects chronic, severe iron deficiency anemia based on which finding?

A

Low ferritin level concentration
Explanation:
The most consistent indicator of iron deficiency anemia is a low ferritin level, which reflects low iron stores. As the anemia progresses, the MCV, which measures the size of the erythrocytes, also decreases. Hematocrit and RBC levels are also low in relation to the hemoglobin concentration.

How well did you know this?
1
Not at all
2
3
4
5
Perfectly
62
Q

The nurse is instructing the client with sickle cell disease about the use of an inhaled vasodilator that may reduce sickling. What medication is the nurse instructing the client about?

A

Nitric oxide
Explanation:
Inhaled nitric oxide—not nitrous oxide (laughing gas), a vasodilating agent—is believed to reduce sickling by promoting the binding of oxygen to hemoglobin. It is being used in the form of handheld inhalers to abort or relieve pain experienced during sickle cell crises. Betamethasone is a corticosteroid, and terbutaline is not used as an inhaler.

How well did you know this?
1
Not at all
2
3
4
5
Perfectly
63
Q

A nurse should expect to administer which vaccine to the client after a splenectomy?

A

Pneumovax 23
Explanation:
Pneumovax 23, a polyvalent pneumococcal vaccine, is administered prophylactically to prevent the pneumococcal sepsis that sometimes occurs after splenectomy. Recombivax HB is a vaccine for hepatitis B. Attenuvax is a live, attenuated virus vaccine for immunization against measles (rubeola). Tetanus toxoid is administered to prevent tetanus resulting from impaired skin integrity caused by traumatic injury.

How well did you know this?
1
Not at all
2
3
4
5
Perfectly
64
Q

The nurse provides care for an older adult client, diagnosed with anemia, who has a hemoglobin of 9.6 g/dL and a hematocrit of 34%. To determine the cause of the client’s blood loss, which is the priority nursing action?

A

Observe the client’s stools for blood.
Explanation:
If an older adult is anemic, blood loss from the gastrointestinal (GI) or genitourinary (GU) tracts is suspected. Observing the stool for blood will determine if the source of the client’s bleeding is in the GI tract. Iron-deficiency anemia is unusual in older adults because the body does not eliminate excessive iron, thus increasing total body iron stores and necessitating maintenance of hydration. If evaluation of the GI and GU tracts does not reveal a source of bleeding, evaluating the client’s diet may be appropriate; however, this is not the priority nursing action. Monitoring the client’s body temperature and BP will assist the nurse in determining the source of the client’s blood loss, but these are not priority nursing actions.

How well did you know this?
1
Not at all
2
3
4
5
Perfectly
65
Q

The nurse is planning care for a client with severe fatigue secondary to anemia. What concept will the nurse use as the basis for planning interventions?

A

Assisting in prioritizing activities.
Explanation:
When planning care for a client with severe fatigue secondary to anemia, the nurse should act collaboratively with the client and assist in prioritizing activities. The client ultimately determines the balance between rest and activity, not the nurse. The nurse will balance activities and group nursing interventions in order to prevent client fatigue.

How well did you know this?
1
Not at all
2
3
4
5
Perfectly
66
Q

A client is prescribed 325 mg/day of oral ferrous sulfate. What does the nurse include in client teaching?

A

Take 1 hour before breakfast
Explanation:
Instructions the nurse will provide for the client taking oral ferrous sulfate is to administer the medication on an empty stomach. Instructions also include that there is decreased absorption of iron with food, particularly dairy products. The client is to increase vitamin C intake (fruits, juices, tomatoes, broccoli), which will enhance iron absorption. The client is to also increase foods high in fiber to decrease risk of constipation.

How well did you know this?
1
Not at all
2
3
4
5
Perfectly
67
Q

A nurse is doing a physical examination of a child with sickle cell anemia. When the child asks why the nurse auscultates the lungs and heart, what would be best the response by the nurse?

A

To detect the abnormal sounds suggestive of acute chest syndrome and heart failure
Explanation:
The nurse auscultates the lungs and heart to detect abnormal sounds that indicate pneumonia, acute chest syndrome, and heart failure. The nurse assesses vital signs to detect evidence of infection, such as fever and tachycardia. During the physical examination, the nurse observes the client’s appearance, looking for evidence of dehydration, which may have triggered a sickle cell crisis. The nurse assesses mental status, verbal ability, and motor strength to detect stroke-related signs and symptoms.

How well did you know this?
1
Not at all
2
3
4
5
Perfectly
68
Q

A nurse is caring for a client with thrombocytopenia. What is the best way to protect this client?

A

Use the smallest needle possible for injections.
Explanation:
Because thrombocytopenia alters coagulation, it poses a high risk of bleeding. To help prevent capillary bleeding, the nurse should use the smallest needle possible when administering injections. The nurse doesn’t need to limit visits by family members because they don’t pose any danger to the client. The nurse should provide comfort measures and maintain the client on bed rest; activities such as using a wheelchair can cause bleeding. The nurse records fluid intake and output to monitor hydration; however, this action doesn’t protect the client from a complication of thrombocytopenia.

How well did you know this?
1
Not at all
2
3
4
5
Perfectly
69
Q

A male client has been receiving a continuous infusion of weight–based heparin for more than 4 days. The client’s PTT is at a level that requires an increase of heparin by 100 units per hour. The client has the laboratory findings shown above. What is the most important action for the nurse to take?

A

Consult with the physician about discontinuing heparin.
Explanation:
Platelet counts may decrease with heparin therapy, and this client’s platelet count has decreased. The client may have heparin–induced thrombocytopenia (HIT). Treatment of HIT includes discontinuing the heparin. The question asks about the most important action of the nurse and that is to consult with the physician about discontinuing heparin therapy. The nurse may continue with the current rate and should not increase the heparin dose until consulting with the physician. Warfarin is not administered until the platelet count has returned to normal levels.

How well did you know this?
1
Not at all
2
3
4
5
Perfectly
70
Q

Which nursing intervention should be incorporated into the plan of care to manage the delayed clotting process in a client with leukemia?

A

Apply prolonged pressure to needle sites or other sources of external bleeding
Explanation:
For a client with leukemia, the nurse should apply prolonged pressure to needle sites or other sources of external bleeding. Reduced platelet production results in a delayed clotting process and increases the potential for hemorrhage. Implementing neutropenic precautions and eliminating direct contact with others are interventions to address the risk for infection.

How well did you know this?
1
Not at all
2
3
4
5
Perfectly
71
Q

A female client with the beta-thalassemia trait plans to marry a man of Italian ancestry who also has the trait. Which client statement indicates that she understands the teaching provided by the nurse?

A

“I’ll see a genetic counselor before starting a family.”
Explanation:
Two people with the beta-thalassemia trait have a 25% chance of having a child with thalassemia major, a potentially life-threatening disease. Iron supplements aren’t used to treat thalassemia; in fact, they could contribute to iron overload. Vitamin B<!sub>12!sub> injections are used to treat pernicious anemia, not thalassemia. Thalassemia occurs primarily in people of Italian, Greek, African, Asian, Middle Eastern, East Indian, and Caribbean descent.

How well did you know this?
1
Not at all
2
3
4
5
Perfectly
72
Q

An older adult client who is a vegetarian has a hemoglobin of 10.2 gm/dL, vitamin B12 of 68 pg/mL (normal: 200–900 pg/mL), and MCV of 110 cubic micrometers. After interpreting the data, what instruction should the nurse give to the client?

A

Supplement the diet with vitamin B12.
Explanation:
Data support that the client is experiencing megaloblastic anemia. Findings include the laboratory test results, the client’s older age, and the client’s status as a vegetarian. Many vegetarians need to supplement their diet with vitamin B12. Eating more foods with vitamin B12 will not provide enough of this vitamin for the client’s body. Increasing iron sources will not resolve the client’s anemia. Telling the client to discontinue the vegetarian practice and eat red meat is nontherapeutic.

How well did you know this?
1
Not at all
2
3
4
5
Perfectly
73
Q

A client with a pulmonary embolism is being treated with a heparin infusion. What diagnostic finding suggests to the nurse that treatment is effective?

A

The client’s activated partial thromboplastin time (aPTT) is 1.5 to 2.5 times the control value.
Explanation:
The therapeutic effect of heparin is monitored by serial measurements of the aPTT; the dose is adjusted to maintain the range at 1.5 to 2.5 times the laboratory control. Heparin dosing is not determined on the basis of platelet levels, the presence or absence of clotting factors, or PT levels.

How well did you know this?
1
Not at all
2
3
4
5
Perfectly
74
Q

After receiving chemotherapy for lung cancer, a client’s platelet count falls to 98,000/mm3. What term should the nurse use to describe this low platelet count?

A

Thrombocytopenia
Explanation:
A normal platelet count is 140,000 to 400,000/mm3 in adults. Chemotherapeutic agents produce bone marrow depression, resulting in reduced red blood cell counts (anemia), reduced white blood cell counts (leukopenia), and reduced platelet counts (thrombocytopenia). Neutropenia is the presence of an abnormally reduced number of neutrophils in the blood and is caused by bone marrow depression induced by chemotherapeutic agents.

How well did you know this?
1
Not at all
2
3
4
5
Perfectly
75
Q

A client with sickle cell anemia has a

A

low hematocrit.
Explanation:
A client with sickle cell anemia has a low hematocrit and sickled cells on the smear. A client with sickle cell trait usually has a normal hemoglobin level, a normal hematocrit, and a normal blood smear.

How well did you know this?
1
Not at all
2
3
4
5
Perfectly
76
Q

A clinical nurse specialist (CNS) is orienting a new graduate registered nurse to an oncology unit where blood product transfusions are frequently administered. In discussing ABO compatibility, the CNS presents several hypothetical scenarios. The new graduate knows that the greatest likelihood of an acute hemolytic reaction would occur when giving:

A

A-positive blood to an A-negative client.
Explanation:
An acute hemolytic reaction occurs when there is an ABO or Rh incompatibility. For example, giving A blood to a B client would cause a hemolytic reaction. Likewise, giving Rh-positive blood to an Rh-negative client would cause a hemolytic reaction. It’s safe to give Rh-negative blood to an Rh-positive client if there is a blood type compatibility. O-negative blood is the universal donor and can be given to all other blood types. AB clients can receive either A or B blood as long as there isn’t an Rh incompatibility.

77
Q

A few minutes after beginning a blood transfusion, a nurse notes that a client has chills, dyspnea, and urticaria. The nurse reports this to the health care provider immediately because the client probably is experiencing which problem?

A

A hemolytic allergic reaction caused by an antigen reaction
Explanation:
Hemolytic allergic reactions are fairly common and may cause chills, fever, urticaria, tachycardia, dyspnea, chest pain, hypotension, and other signs of anaphylaxis a few minutes after blood transfusion begins. Although rare, a hemolytic reaction to mismatched blood can occur, triggering a more severe reaction and, possibly, leading to disseminated intravascular coagulation. A hemolytic reaction to Rh-incompatible blood is less severe and occurs several days to 2 weeks after the transfusion. Bacterial contamination of donor blood causes a high fever, nausea, vomiting, diarrhea, abdominal cramps and, possibly, shock.

78
Q

Which of the following is considered an antidote to heparin?

A

Protamine sulfate
Explanation:
Protamine sulfate, in the appropriate dosage, acts quickly to reverse the effects of heparin. Vitamin K is the antidote to warfarin (Coumadin). Narcan is the drug used to reverse signs and symptoms of medication-induced narcosis. Ipecac is an emetic used to treat some poisonings.

79
Q

When teaching a client with iron deficiency anemia about appropriate food choices, the nurse encourages the client to increase the dietary intake of which foods?

A

Beans, dried fruits, and leafy, green vegetables
Explanation:
Food sources high in iron include organ meats (e.g., beef or calf liver, chicken liver), other meats, beans (e.g., black, pinto, and garbanzo), leafy and green vegetables, raisins, and molasses. Taking iron-rich foods with a source of vitamin C (e.g., orange juice) enhances the absorption of iron.

80
Q

A client in end–stage renal disease is prescribed epoetin alfa and oral iron supplements. Before administering the next dose of epoetin alfa and oral iron supplement, what is the priority action taken by the nurse?

A

Assesses the hemoglobin level
Explanation:
Erythropoietin (epoetin alfa [Epogen]) with oral iron supplements can raise hematocrit levels in the client with end–stage renal disease. The nurse should check the hemoglobin prior to administration of erythropoietin, because too high a hemoglobin level can put the client at risk for heart failure, myocardial infarction, and cerebrovascular accident. Erythropoietin may be administered during dialysis treatments. The BUN will be elevated in the client with end–stage renal disease.

81
Q

The nurse, caring for a client in the emergency room with a severe nosebleed, becomes concerned when the client asks for a bedpan. The nurse documents the stool as loose, tarry, and black looking. The nurse suspects the client may have thrombocytopenia. What should be the nurse’s priority action?

A

Notify the physician
Explanation:
Thrombocytopenia is evidenced by purpura, small hemorrhages in the skin, mucous membranes, or subcutaneous tissues. Bleeding from other parts of the body, such as the nose, oral mucous membrane, and the gastrointestinal tract, also occurs. Internal hemorrhage, which can be severe and even fatal, is possible. This nurse should notify the physician of the suspected disorder.

82
Q

Which term refers to an abnormal decrease in white blood cells, red blood cells, and platelets?

A

Pancytopenia
Explanation:
Pancytopenia is defined as an abnormal decrease in WBCs, RBCs, and platelets. The condition may be congenital or acquired. Anemia refers to decreased red cell mass. Leukopenia refers to a less-than-normal amount of WBCs in circulation. Thrombocytopenia refers to a lower-than-normal platelet count.

83
Q

A client admitted to the hospital in preparation for a splenectomy to treat autoimmune hemolytic anemia asks the nurse about the benefits of splenectomy. Which statement best explains the expected effect of splenectomy?

A

It will remove the major site of red blood cell (RBC) destruction.
Explanation:
For clients with autoimmune hemolytic anemia, if corticosteroids do not produce remission, a splenectomy (i.e., removal of the spleen) may be performed because it removes the major site of RBC destruction.

84
Q

A client with multiple myeloma is complaining of severe pain when the nurse comes in to give a bath and change position. What is the priority intervention by the nurse?

A

Obtain the pain medication and delay the bath and position change until the medication reaches its peak.
Explanation:
When pain is severe, the nurse delays position changes and bathing until an administered analgesic has reached its peak concentration level and the client is experiencing maximum pain relief. Pain medication should never be delayed to assist in the control of the level of pain. Pain will not be relieved by a bath and clean sheets, only analgesics at this point in the client’s illness.

85
Q

A health care provider prescribes one tablet of ferrous sulfate daily for a 15-year-old girl who experiences heavy blood flow during her menstrual cycle. The nurse advises the patient and her parent that this over-the-counter preparation must be taken for how many months before stored iron replenishment can occur?

A

6 to 12 months
Explanation:
Ferrous sulfate can increase hemoglobin levels in a few weeks, and anemia may be corrected in a few months. However, it takes 6 to 12 months for stored iron replenishment to occur.

86
Q

A patient describes numbness in the arms and hands with a tingling sensation. The patient also frequently stumbles when walking. What vitamin deficiency does the nurse determine may cause some of these symptoms?

A

B12
Explanation:
The hematologic effects of vitamin B12 deficiency are accompanied by effects on other organ systems, particularly the gastrointestinal tract and nervous system. Patients with pernicious anemia may become confused; more often, they have paresthesias in the extremities (particularly numbness and tingling in the feet and lower legs). They may have difficulty maintaining their balance because of damage to the spinal cord, and they also lose position sense (proprioception).

87
Q

A nurse is caring for a client with thalassemia who is being transfused. What is the nurse’s role during a transfusion?

A

To closely monitor the rate of administration
Explanation:
In a client with thalassemia, when transfusions are necessary, the nurse closely monitors the rate of administration. Assessing for enlargement and tenderness over the liver and spleen, advising rest, or administering vitamin B12 injections are not indicated for thalassemia.

88
Q

A client is being treated for DIC and the nurse has prioritized the nursing diagnosis of Risk for Deficient Fluid Volume Related to Bleeding. How can the nurse best determine if goals of care relating to this diagnosis are being met?

A

Closely monitor intake and output.
Explanation:
The client with DIC is at a high risk of deficient fluid volume. The nurse can best gauge the effectiveness of care by closely monitoring the client’s intake and output. Each of the other assessments is a necessary element of care, but none addresses fluid balance as directly as close monitoring of intake and output.

89
Q

While assessing a client, the nurse discovers the client has a history of restless leg syndrome. Which hematological condition does the nurse associate with this condition?

A

Iron deficiency anemia
Explanation:
Restless leg syndrome is common in as many as 24% of those with iron deficiency anemia.

90
Q

After receiving chemotherapy for lung cancer, a client’s platelet count falls to 98,000/mm3. What term should the nurse use to describe this low platelet count?

A

Thrombocytopenia
Explanation:
A normal platelet count is 140,000 to 400,000/mm3 in adults. Chemotherapeutic agents produce bone marrow depression, resulting in reduced red blood cell counts (anemia), reduced white blood cell counts (leukopenia), and reduced platelet counts (thrombocytopenia). Neutropenia is the presence of an abnormally reduced number of neutrophils in the blood and is caused by bone marrow depression induced by chemotherapeutic agents.

91
Q

Clinical manifestations of common bile duct obstruction include all of the following except:

A

Light-colored urine
Explanation:
The excretion of the bile pigments by the kidneys gives the urine a very dark color. The feces, no longer colored with bile pigments, are grayish, like putty, or clay-colored. The symptoms may be acute or chronic. Epigastric distress, such as fullness, abdominal distention, and vague pain in the right upper quadrant of the abdomen, may occur. If it goes untreated jaundice and pruritus can occur.

92
Q

The digestion of carbohydrates is aided by

A

amylase.
Explanation:
Amylase is secreted by the exocrine pancreas. Lipase aids in the digestion of fats. Trypsin aids in the digestion of proteins. Secretin is the major stimulus for increased bicarbonate secretion from the pancreas.

93
Q

A client being treated for pancreatitis faces the risk of atelectasis. Which of the following interventions would be important to implement to minimize this risk?

A

Use incentive spirometry every hour.
Explanation:
The nurse instructs the client in techniques of coughing and deep breathing and in the use of incentive spirometry to improve respiratory function. The nurse assists the client to perform these activities every hour. Repositioning the client every 2 hours minimizes the risk of atelectasis. The client should be instructed to cough every 2 hours to reduce atelectasis. Monitoring pulse oximetry helps show changes in respiratory status and promotes early intervention, but it would do little to minimize the risk of atelectasis. Withholding analgesics is not an appropriate intervention due to the severe pain associated with pancreatitis.

94
Q

A client is admitted to the health care facility with abdominal pain, a low-grade fever, abdominal distention, and weight loss. The physician diagnoses acute pancreatitis. What is the primary goal of nursing care for this client?

A

Relieving abdominal pain
Explanation:
The predominant clinical feature of acute pancreatitis is abdominal pain, which usually reaches peak intensity several hours after onset of the illness. Therefore, relieving abdominal pain is the nurse’s primary goal. Because acute pancreatitis causes nausea and vomiting, the nurse should try to prevent fluid volume deficit, not overload. The nurse can’t help the client achieve adequate nutrition or understand the disease and its treatment until the client is comfortable and no longer in pain.

95
Q

A client with gallstones is diagnosed with acute pancreatitis and is requesting information about the physiology of the gallbladder. Which information will the nurse include about the function of this organ?

A

Releases bile in response to cholecystokinin

Explanation:
The gallbladder is a pear-shaped, hollow, saclike organ that lies in a shallow depression on the inferior surface of the liver. When food enters the duodenum, the gallbladder contracts and the sphincter of Oddi relaxes. Relaxation of this sphincter allows the bile to enter the intestine. This response is mediated by secretion of the hormone cholecystokinin (CCK) from the intestinal wall. Gallstones can block the bile duct and digestive juices to the pancreas causing acute pancreatitis. The gallbladder functions as a storage depot for bile. Bile does not digest carbohydrates in the jejunum. The liver controls the flow of trypsin to digest proteins.

96
Q

A client with acute pancreatitis has been started on total parenteral nutrition (TPN). Which action should the nurse perform after administration of the TPN?

A

Measure blood glucose concentration every 4 to 6 hours
Explanation:
Enteral or parenteral nutrition may be prescribed. In addition to administering enteral or parenteral nutrition, the nurse monitors the serum glucose concentration every 4 to 6 hours.

97
Q

When the nurse is caring for a patient with acute pancreatitis, what intervention can be provided in order to prevent atelectasis and prevent pooling of respiratory secretions?

A

Frequent changes of positions
Explanation:
Frequent changes of position are necessary to prevent atelectasis and pooling of respiratory secretions.

98
Q

Upon receiving the dinner tray for a client admitted with acute gallbladder inflammation, the nurse will question which of the following foods on the tray?

A

Hot roast beef sandwich with gravy
Explanation:
The diet immediately after an episode of acute cholecystitis is initially limited to low-fat liquids. Cooked fruits, rice or tapioca, lean meats, mashed potatoes, bread, and coffee or tea may be added as tolerated. The client should avoid fried foods such as roast beef because fatty foods may bring on an episode of cholecystitis.

99
Q

A client with cholelithiasis has a gallstone lodged in the common bile duct. When assessing this client, the nurse expects to note:

A

yellow sclerae.
Explanation:
Yellow sclerae are an early sign of jaundice, which occurs when the common bile duct is obstructed. Urine normally is light amber. Circumoral pallor and black, tarry stools don’t occur in common bile duct obstruction; they are signs of hypoxia and GI bleeding, respectively.

100
Q

Which condition in a client with pancreatitis makes it necessary for the nurse to check fluid intake and output, check hourly urine output, and monitor electrolyte levels?

A

Frequent vomiting, leading to loss of fluid volume
Explanation:
Fluid and electrolyte disturbances are common complications because of nausea, vomiting, movement of fluid from the vascular compartment to the peritoneal cavity, diaphoresis, fever, and the use of gastric suction. The nurse assesses the client’s fluid and electrolyte status by noting skin turgor and moistness of mucous membranes. The nurse weighs the client daily and carefully measures fluid intake and output, including urine output, nasogastric secretions, and diarrhea.

101
Q

Which enzyme aids in the digestion of fats?

A

Lipase
Explanation:
Lipase is a pancreatic enzyme that aids in the digestion of fats. Amylase aids in the digestion of carbohydrates. Secretin is responsible for stimulating secretion of pancreatic juice. Trypsin aids in the digestion of protein.

102
Q

A student nurse is preparing a plan of care for a client with chronic pancreatitis. What nursing diagnosis related to the care of a client with chronic pancreatitis is the priority?

A

Impaired nutrition: less than body requirements
Explanation:
While each diagnosis may be applicable to this client, the priority nursing diagnosis is impaired nutrition: less than body requirements. The physician, nurse, and dietitian emphasize to the client and family the importance of avoiding alcohol and foods that have produced abdominal pain and discomfort in the past. Oral food or fluid intake is not permitted during the acute phase.

103
Q

A patient is diagnosed with mild acute pancreatitis. What does the nurse understand is characteristic of this disorder?

A

Edema and inflammation
Explanation:
Mild acute pancreatitis is characterized by edema and inflammation confined to the pancreas. Minimal organ dysfunction is present, and return to normal function usually occurs within 6 months.

104
Q

A client with a history of alcohol abuse comes to the emergency department and complains of abdominal pain. Laboratory studies help confirm a diagnosis of acute pancreatitis. The client’s vital signs are stable, but the client’s pain is worsening and radiating to his back. Which intervention takes priority for this client?

A

Administering morphine I.V. as ordered
Explanation:
The nurse should address the client’s pain issues first by administering morphine I.V. as ordered. Placing the client in a Semi-Fowler’s position, maintaining NPO status, and providing mouth care don’t take priority over addressing the client’s pain issues.

105
Q

Total parental nutrition (TPN) should be used cautiously in clients with pancreatitis because such clients:

A

cannot tolerate high-glucose concentration.
Explanation:
Total parental nutrition (TPN) is used carefully in clients with pancreatitis because some clients cannot tolerate a high-glucose concentration even with insulin coverage. Intake of coffee increases the risk for gallbladder contraction, whereas intake of high protein increases risk for hepatic encephalopathy in clients with cirrhosis. Patients with pancreatitis should not be given high-fat foods because they are difficult to digest.

106
Q

The nurse is planning care for a client following an incisional cholecystectomy for cholelithiasis. Which intervention is the highest nursing priority for this client?

A

Assisting the client to turn, cough, and deep breathe every 2 hours
Explanation:
Assessment should focus on the client’s respiratory status. If a traditional surgical approach is planned, the high abdominal incision required during surgery may interfere with full respiratory excursion. The other nursing actions are also important, but are not as high a priority as ensuring adequate ventilation.

107
Q

A nurse is caring for a client in a critical care unit. With what type of shock does a client experience a pooling of blood flow to the peripheral blood vessels?

A

distributive
Explanation:
Distributive shock results from displacement of blood volume, creating pooling of blood in the peripheral blood vessels. Cardiogenic shock results from the failure of a heart as a pump. With hypovolemic shock, there is a decrease in the intravascular volume. Organ failure is not a type of shock.

108
Q

The nurse knows when the cardiovascular system becomes ineffective in maintaining an adequate mean arterial pressure (MAP). Select the reading below that indicates tissue hypoperfusion.

A

60 mm Hg
Explanation:
Mean arterial pressure is cardiac output × peripheral resistance. The body must exceed 65 mm Hg MAP for cells to receive oxygen and nutrients.

109
Q

A nurse is assisting with the orientation of a newly hired graduate. Which of the following behaviors of the graduate nurse would the other nurse identify as not adhering to strict infection control practices?

A

Hanging tape on the bedside table when changing a wet–to–dry sterile dressing
Explanation:
The Centers for Disease Control and Prevention do not recommend hanging tape on bedside tables, siderails, linens, or clothing to use for dressings. The other options are activities that are proper infection control practices.

110
Q

When a client is in the compensatory stage of shock, which symptom occurs?

A

Tachycardia
Explanation:
The compensatory stage of shock encompasses a normal BP, tachycardia, decreased urinary output, confusion, and respiratory alkalosis.

111
Q

The nurse is caring for a motor vehicle accident client who is unresponsive on arrival to the emergency department. The client has numerous fractures, internal abdominal injuries, and large lacerations on the head and torso. The family arrives and seeks update on the client’s condition. A family member asks, “What causes the body to go into shock?” Given the client’s condition, which statement is most correct?

A

“The client is in shock because the blood volume has decreased in the system.”
Explanation:
Shock is a life-threatening condition that occurs when arterial blood flow and oxygen delivery to tissues and cells are inadequate. Hypovolemic shock, where the volume of extracellular fluid is significantly diminished due to the loss of or reduced blood or plasma, frequently occurs with accidents.

112
Q

Which colloid is expensive but rapidly expands plasma volume?

A

Albumin
Explanation:
Albumin is a colloid that requires human donors, is limited in supply, and can cause congestive heart failure. Dextran interferes with platelet aggregation and is not recommended for hemorrhagic shock. Lactated Ringer solution and hypertonic saline are crystalloids, not colloids.

113
Q

A patient arrives in the emergency department with complaints of chest pain radiating to the jaw. What medication does the nurse anticipate administering to reduce pain and anxiety as well as reducing oxygen consumption?

A

Correct response:
Morphine
Explanation:
If a patient experiences chest pain, IV morphine is administered for pain relief. In addition to relieving pain, morphine dilates the blood vessels. This reduces the workload of the heart by both decreasing the cardiac filling pressure (preload) and reducing the pressure against which the heart muscle has to eject blood (afterload). Morphine also decreases the patient’s anxiety and reduces the respiratory rate, and thus oxygen consumption.

114
Q

The nurse, a member of the health care team in the ED, is caring for a client who is determined to be in the irreversible stage of shock. What would be the most appropriate nursing intervention?

A

Provide opportunities for the family to spend time with the client, and help them to understand the irreversible stage of shock.
Explanation:
The irreversible (or refractory) stage of shock represents the point along the shock continuum at which organ damage is so severe that the client does not respond to treatment and cannot survive. Providing opportunities for the family to spend time with the client and helping them to understand the irreversible stage of shock is the best intervention. Informing the client’s family early that the client will likely not survive does allow the family to make plans and move forward, but informing the family too early will rob the family of hope and interrupt the grieving process. The chance of surviving the irreversible (or refractory) stage of shock is very small, and the nurse needs to help the family cope with the reality of the situation. With the chances of survival so small, the priorities shift from aggressive treatment and safety to addressing the end-of-life issues.

115
Q

Shock occurs when tissue perfusion is inadequate to deliver oxygen and nutrients to support cellular function. When caring for patients who may develop indicators of shock, the nurse is aware that the most important measurement of shock is:

A

Blood pressure.
Explanation:
By the time the blood pressure drops, damage has already been occurring at the cellular and tissue levels. Therefore, the patient at risk for shock must be monitored closely before the blood pressure drops.

116
Q

A patient visits a health clinic because of urticaria and shortness of breath after being stung by several wasps. The nurse practitioner immediately administers which medication to reduce bronchospasm?

A

Epinephrine
Explanation:
Epinephrine is given for its vasoconstrictive actions, as well as for its rapid effect of reducing bronchospasm. Benadryl and Proventil (nebulized) are given to reverse the effects of histamine. Prednisone is given to reduce inflammation, if necessary.

117
Q

The nurse receives an order to administer a colloidal solution for a patient experiencing hypovolemic shock. What common colloidal solution will the nurse most likely administer?

A

5% albumin
Explanation:
Typically, if colloids are used to treat tissue hypoperfusion, albumin is the agent prescribed. Albumin is a plasma protein; an albumin solution is prepared from human plasma and is heated during production to reduce its potential to transmit disease. The disadvantage of albumin is its high cost compared to crystalloid solutions. Hetastarch and dextran solutions are not indicated for fluid administration because these agents interfere with platelet aggregation. Blood products are not indicated in this situation.

118
Q

The nurse determines that a patient in shock is experiencing a decrease in stroke volume when what clinical manifestation is observed?

A

Narrowed pulse pressure
Explanation:
Pulse pressure correlates well with stroke volume. Pulse pressure is calculated by subtracting the diastolic measurement from the systolic measurement; the difference is the pulse pressure. Normally, the pulse pressure is 30 to 40 mm Hg. Narrowing or decreased pulse pressure is an earlier indicator of shock than a drop in systolic BP. Decreased or narrowing pulse pressure is an early indication of decreased stroke volume.

119
Q

The nursing instructor is discussing shock with the senior nursing students. The instructor tells the students that shock is a life-threatening condition. What else should the instructor tell the students about shock?

A

It occurs when arterial blood flow and oxygen delivery to tissues and cells are inadequate.
Explanation:
Shock is a life-threatening condition that occurs when arterial blood flow and oxygen delivery to tissues and cells are inadequate. Respiratory distress syndrome can be a complication of shock but is not necessarily caused by shock. Shock does not begin when peripheral blood flow is inadequate. Not every trauma victim goes into shock.

120
Q

A large volume of intravenous fluids is being administered to an elderly client who experienced hypovolemic shock following diarrhea. The nurse is evaluating the client’s response to treatment and notes the following as a sign of an adverse reaction:

A

Jugular venous distention
Explanation:
When administering large volumes of fluid replacement, the nurse monitors the client for cardiovascular overload, signs of difficulty breathing, and pulmonary edema. The nurse assesses for jugular vein distention. Decreased pulse rate, when the client is tachycardic as in hypovolemic shock, would indicate improvement. The client would also exhibit a positive increase in the fluid balance ratio when responding appropriately to treatment. The client should exhibit vesicular breath sounds.

121
Q

A patient is in the progressive stage of shock with lung decompensation. What treatment does the nurse anticipate assisting with?

A

Intubation and mechanical ventilation
Explanation:
Decompensation of the lungs increases the likelihood that mechanical ventilation will be needed. Administration of oxygen via a mask would be appropriate in the compensatory stage but insufficient in the event of lung decompensation. Pericardiocentesis or thoracotomy with chest tube insertion would not be necessary or appropriate.

122
Q

Morphine sulfate has which of the following effects on the body?

A

Reduces preload
Explanation:
In addition to relieving pain, morphine dilates the blood vessels. This reduces the workload of the heart by both decreasing the cardiac filing pressure (preload) and reducing the pressure against which the heart muscle has to eject blood (afterload).

123
Q

A client experiences an acute myocardial infarction. Current blood pressure is 90/58, pulse is 118 beats/minute, and respirations are 30 breaths/minute. The nurse intervenes first by administering the following prescribed treatment:

A

Oxygen at 2 L/min by nasal cannula
Explanation:
In the early stages of cardiogenic shock, the nurse first administers supplemental oxygen to achieve an oxygen saturation exceeding 90%. The nurse may then administer morphine to relieve chest pain and/or to reduce the workload of the heart and decrease client anxiety. Intravenous fluids are given carefully to prevent fluid overload. Vasoactive medications, such as dopamine, are then administered to restore and maintain cardiac output.

124
Q

The nurse is reviewing diagnostic lab work of a client developing shock. Which laboratory result does the nurse note as a key in determining the type of shock?

A

WBC: 42,000/mm3
Explanation:
Septic shock has the highest mortality rate and is caused by an overwhelming bacterial infection; thus, an elevated WBC can indicate this type of shock. The other lab values are within normal limits.

125
Q

You are the nurse caring for a client in septic shock. You know to closely monitor your client. What finding would you observe when the client’s condition is in its initial stages?

A

A rapid, bounding pulse
Explanation:
A rapid, bounding pulse is observed in a client in the initial stages of septic shock. In case of hypovolemic shock, the pulse volume becomes weak and thready and circulating volume diminishes in the initial stage. In the later stages when the circulating volume has severely diminished, the pulse becomes slow and imperceptible and pulse rhythm changes from regular to irregular.

126
Q

The nurse is administering a medication to the client with a positive inotropic effect. Which action of the medication does the nurse anticipate?

A

Increase the force of myocardial contraction
Explanation:
The nurse realizes that when administering a medication with a positive inotropic effect, the medication increases the force of heart muscle contraction. The heart rate increases not decreases. The central nervous system is not depressed nor is there a dilation of the bronchial tree.

127
Q

A nurse is providing care to all of the following clients. Which would be at increased risk for anaphylactic shock? Select all that apply.

A

Risk factors for anaphylactic shock include transfusion reaction, latex allergy, and severe allergy to foods or medications. The client in the first 15 minutes of receiving blood is at risk for an anaphylactic reaction. This is why the nurse should remain in the room for the first 15 minutes of infusion. The client with spina bifida is at risk for a latex allergy, which, in turn, increases the risk for an anaphylactic reaction if latex gloves are used. The client with a peanut allergy is at risk for an anaphylactic reaction if food is prepared or accidentally contaminated with a nut–based oil. The other clients are not at an increased risk for anaphylactic shock.

128
Q

A client has experienced hypovolemic shock and is being treated with 2 liters of lactated Ringer’s solution. It is now most important for the nurse to assess

A

Lung sounds
Explanation:
The nurse must monitor the client during fluid replacement for side effects and complications. The most common and serious side effects include cardiovascular overload and pulmonary edema, which would be exhibited as adventitious lung sounds. Other assessments that the nurse would make include skin perfusion, changes in mentation, and bowel sounds.

129
Q

You are caring for a client who is in neurogenic shock. You know that this is a subcategory of what kind of shock?

A

Circulatory (distributive)
Explanation:
Three types of circulatory (distributive) shock are neurogenic, septic, and anaphylactic shock. There is no such thing as carcinogenic shock. Obstructive and hypovolemic shock do not have subcategories.

130
Q

The nurse is caring for a client in shock who is deteriorating. The nurse is infusing IV fluids and giving medications as ordered. What type of medications is the nurse most likely giving to this client?

A

Adrenergic drugs
Explanation:
Adrenergic drugs are the main medications used to treat shock due to their action on the receptors of the sympathetic nervous system.

131
Q

The central venous pressure (CVP) reading in hypovolemic shock is typically which of the following?

A

Low
Explanation:
The CVP reading is typically low in hypovolemic shock. It increases with effective treatment and is significantly increased with fluid overload and heart failure.

132
Q

A client experiencing vomiting and diarrhea for 2 days has a blood pressure of 88/56, a pulse rate of 122 beats/minute, and a respiratory rate of 28 breaths/minute. The nurse places the client in which position?

A

Modified Trendelenburg
Explanation:
The client is experiencing hypovolemic shock as a result of prolonged vomiting and diarrhea. The modified Trendelenburg position is recommended for hypovolemic shock because it promotes the return of venous blood. The other positions may make breathing difficult and may not increase blood pressure or cardiac output.

133
Q

The nurse is monitoring the patient in shock. The patient begins bleeding from previous venipuncture sites, in the indwelling catheter, and rectum, and the nurse observes multiple areas of ecchymosis. What does the nurse suspect has developed in this patient?

A

Disseminated intravascular coagulation (DIC)
Explanation:
Disseminated intravascular coagulation (DIC) may occur either as a cause or as a complication of shock. In this condition, widespread clotting and bleeding occur simultaneously. Bruises (ecchymoses) and bleeding (petechiae) may appear in the skin. Coagulation times (e.g., prothrombin time [PT], activated partial thromboplastin time [aPTT]) are prolonged. Clotting factors and platelets are consumed and require replacement therapy to achieve hemostasis. The other conditions listed would not result in bleeding simultaneously at multiple sites.

134
Q

The nurse is using continuous central venous oximetry (ScvO2) to monitor the blood oxygen saturation of a patient in shock. What value would the nurse document as normal for the patient?

A

70%
Explanation:
Continuous central venous oximetry (ScvO2) monitoring may be used to evaluate mixed venous blood oxygen saturation and severity of tissue hypoperfusion states. A central catheter is introduced into the superior vena cava (SVC), and a sensor on the catheter measures the oxygen saturation of the blood in the SVC as blood returns to the heart and pulmonary system for re-oxygenation. A normal ScvO2 value is 70%.

135
Q

A client with a history of depression is brought to the ED after overdosing on Valium. This client is at risk for developing which type of distributive shock?

A

neurogenic shock
Explanation:
Injury to the spinal cord or head or overdoses of opioids, opiates, tranquilizers, or general anesthetics can cause neurogenic shock. Septic shock is a subcategory of distributive shock, but it is associated with overwhelming bacterial infections. Anaphylactic shock is a subcategory of distributive shock, but it is a severe allergic reaction that follows exposure to a substance to which a person is extremely sensitive, such as bee venom, latex, fish, nuts, and penicillin. Hypovolemic shock is not a subcategory of distributive shock. It occurs when the volume of extracellular fluid is significantly diminished, primarily because of lost or reduced blood or plasma.

136
Q

During preshock, the compensatory stage of shock, the body, through sympathetic nervous system stimulation, will release catecholamines to shunt blood from one organ to another. Which of the following organs will always be protected?

A

Brain
Explanation:
The body displays a “fight-or-flight” response, with the release of catecholamines. Blood will be shunted to the brain, heart, and lungs to ensure adequate blood supply. The organ that will always be protected over the others is the brain.

137
Q

A nurse educator is teaching a group of nurses about assessing critically ill clients for multiple organ dysfunction syndrome (MODS). The nurse educator evaluates understanding by asking the nurses to identify which client would be at highest risk for MODS. It would be the client who is experiencing septic shock and is

A

An older adult man with end-stage renal disease and an infected dialysis access site
Explanation:
MODS may develop when a client experiences septic shock. Those at increased risk for MODS are older clients, clients who are malnourished, and clients with coexisting disease.

138
Q

The nurse is caring for a client newly diagnosed with sepsis. The client has a serum lactate concentration of 6 mmol/L and fluid resuscitation has been initiated. Which value indicates that the client has received adequate fluid resuscitation?

A

Mean arterial pressure of 70 mm Hg
Explanation:
The nurse administers fluids to achieve a target central venous pressure of 8 to 12 mm Hg, mean arterial pressure >65 mm Hg, urine output of 0.5 mL/kg/hr, and an ScvO2 of 70%.

139
Q

Elevating the patient’s legs slightly to improve cerebral circulation is contraindicated in which of the following disease processes?

A

Head injury
Explanation:
An alternative to the “Trendelenburg” position is to elevate the patient’s legs slightly to improve cerebral circulation and promote venous return to the heart, but this position is contraindicated for patients with head injuries.

140
Q

The student nurse is being precepted in the ICU. The student is caring for a client in the compensatory stage of shock who is hypovolemic. Which compensatory mechanism is most important in the re absorption and retention of fluid in the body?

A

Production of antidiuretic hormone and corticosteroid hormones
Explanation:
Antidiuretic hormone (ADH) and corticosteroid hormones play an active role in controlling sodium and water balance. Both ADH and corticosteroid hormones promote fluid re absorption and retention. The renin-angiotensin-aldosterone system is a mechanism that restores blood pressure (BP) when circulating volume is diminished. The release of catecholamines stimulates secretion of epinephrine and norepinephrine.

141
Q

When a client is in the compensatory stage of shock, which symptom occurs?

A

Tachycardia
Explanation:
The compensatory stage of shock encompasses a normal BP, tachycardia, decreased urinary output, confusion, and respiratory alkalosis.

142
Q

What is the major clinical use of dobutamine?

A

increase cardiac output.
Explanation:
Dobutamine (Dobutrex) increases cardiac output for clients with acute heart failure and those undergoing cardiopulmonary bypass surgery. Physicians may use epinephrine hydrochloride, another catecholamine agent, to treat sinus bradycardia. Physicians use many of the catecholamine agents, including epinephrine, isoproterenol, and norepinephrine, to treat acute hypotension. They don’t use catecholamine agents to treat hypertension because catecholamine agents may raise blood pressure.

143
Q

A nurse practitioner visits a patient in a cardiac care unit. She assesses the patient for shock, knowing that the primary cause of cardiogenic shock is:

A

A myocardial infarction.
Explanation:
Cardiogenic shock is seen most frequently as a result of a myocardial infarction.

144
Q

A client experiencing vomiting and diarrhea for 2 days has a blood pressure of 88/56, a pulse rate of 122 beats/minute, and a respiratory rate of 28 breaths/minute. The nurse places the client in which position?

A

Modified Trendelenburg
Explanation:
The client is experiencing hypovolemic shock as a result of prolonged vomiting and diarrhea. The modified Trendelenburg position is recommended for hypovolemic shock because it promotes the return of venous blood. The other positions may make breathing difficult and may not increase blood pressure or cardiac output.

145
Q

A client is experiencing vomiting and diarrhea for 2 days. Blood pressure is 88/56, pulse rate is 122 beats/minute, and respirations are 28 breaths/minute. The nurse starts intravenous fluids. Which of the following prescribed prn medications would the nurse administer next?

A

ondansetron
Explanation:
An antiemetic medication, such as ondansetron (Zofran), is administered for vomiting. It would be administered before loperamide (Imodium) for diarrhea so the client would be able to retain the loperamide. There is no indication that the client requires medication for pain (meperidine [Demerol]) or heartburn (magnesium hydroxide [Maalox]).

146
Q

You are holding a class on shock for the staff nurses at your institution. What would you tell them about the stages of shock?

A

In the compensation stage, catecholamines are released.
Explanation:
Compensatory mechanisms include the release of catecholamines, activation of the renin-angiotensin-aldosterone system, production of antidiuretic and corticosteroid hormones are all mechanisms activated in the compensation stage of shock. Shock does not begin in the decompensation stage.

147
Q

The client exhibits a blood pressure of 110/68 mm Hg, pulse rate of 112 beats/min, temperature of 102°F with skin warm and flushed. Respirations are 30 breaths/min. The nurse assesses the client may be exhibiting the early stage of which shock?

A

Septic
Explanation:
In the early stage of septic shock, the blood pressure may remain normal, the heart rate tachycardic, the respiratory rate increased, and fever with warm, flushed skin. The client, in the other shocks listed, usually present with different signs such as a normal body temperature, hypotension with either tachycardia or bradycardia, skin that is cool and clammy, and respiratory distress.

148
Q

The nurse is caring for a critically ill client. Which of the following is the nurse correct to identify as a positive effect of catecholamine release during the compensation stage of shock?

A

Increase in arterial oxygenation
Explanation:
Catecholamines are neurotransmitters that stimulate responses via the sympathetic nervous system. Catecholamine release increases heart rate and myocardial contraction as well as bronchial dilation improving the efficient exchange of oxygen and carbon dioxide. They do not decrease WBCs or decrease the depressive symptoms. They do not regulate sodium and potassium.

149
Q

A vasoactive medication is prescribed for a patient in shock to help maintain MAP and hemodynamic stability. A medication that acts on the alpha-adrenergic receptors of the SNS is ordered. Its purpose is to:

A

Constrict blood vessels in the cardiorespiratory system.
Explanation:
Alpha- and beta-adrenergic receptors work synergistically to improve hemodynamic stability. Alpha receptors constrict blood vessels in the cardiorespiratory and gastrointestinal systems, as well as in the skin and kidneys.

150
Q

Following a motor vehicle collision, a client is admitted to the emergency department with a blood pressure of 88/46, pulse of 54 beats/min with a regular rhythm, and respirations of 20 breaths/min with clear lung sounds. The client’s skin is dry and warm. The nurse assesses the client to be in which type of shock?

A

Neurogenic
Explanation:
The client in neurogenic shock experiences hypotension, bradycardia, and dry, warm skin. A client experiencing septic shock would exhibit tachycardia. A client in anaphylactic shock would experience respiratory distress. A client in cardiogenic shock would exhibit cardiac dysrhythmias and adventitious lung sounds.

151
Q

A client is admitted to the emergency department after a motorcycle accident. Upon assessment, the client’s vital signs reveal blood pressure of 80/60 mm Hg and heart rate of 145 beats per minute. The client’s skin is cool and clammy. Which medical order for this client will the nurse complete first?

A

100% oxygen via a nonrebreather mask
Explanation:
The management in all types and all phases of shock includes the following: support of the respiratory system with supplemental oxygen and/or mechanical ventilation to provide optimal oxygenation, fluid replacement to restore intravascular volume, vasoactive medications to restore vasomotor tone and improve cardiac function, and nutritional support to address metabolic requirements that are often dramatically increased in shock. The first priority in the initial management of shock is maintenance of the airway and ventilation; thus, 100% oxygen should be applied via a nonrebreather mask. The other orders should be completed after the client’s airway is secure.

152
Q

The nurse is caring for a client in the compensation stage of shock. The nurse knows that one of the body’s mechanisms of compensation in this stage of shock is the renin-angiotensin-aldosterone system. What does this system do?

A

Restores blood pressure
Explanation:
The renin-angiotensin-aldosterone system is a mechanism that restores blood pressure (BP) when circulating volume is diminished. It does not decrease peripheral blood flow, increase catecholamine secretion, or increase the production of antidiuretic hormone.

153
Q

You are a nurse in the Emergency Department (ED) caring for a client presenting with vasodilation. Your assessment indicates that the client’s central blood flow is reduced and their peripheral vascular area is hypervolemic. You notify the physician that this client is in what kind of shock?

A

Circulatory (distributive)
Explanation:
Vasodilatation, a prominent characteristic of circulatory/distributive shock, increases the space in the vascular bed. Central blood flow is reduced because peripheral vascular or interstitial areas exceed their usual capacity. Vasodilation is not a major component of cardiogenic, hypovolemic, or obstructive shock.

154
Q

The nurse obtains a blood pressure of 120/78 mm Hg from a patient in hypovolemic shock. Since the blood pressure is within normal range for this patient, what stage of shock does the nurse realize this patient is experiencing?

A

Compensatory stage
Explanation:
In the compensatory stage of shock, the BP remains within normal limits. Vasoconstriction, increased heart rate, and increased contractility of the heart contribute to maintaining adequate cardiac output. In all other stages of shock, hypotension is present as compensatory mechanisms no longer suffice to maintain normal blood pressure.

155
Q

A client is being cared for in the Neurological Intensive Care Unit following a spinal cord injury. Which assessment finding indicates that the client may be experiencing neurogenic shock?

A

HR, 48 bpm; BP, 90/60 mm Hg
Explanation:
The clinical characteristics of neurogenic shock are signs of parasympathetic stimulation. It is characterized by dry, warm skin rather than the cool, moist skin seen in hypovolemic shock. Another characteristic is hypotension with bradycardia, rather than the tachycardia that characterizes other forms of shock. The other signs and symptoms are associated with hypovolemic shock.

156
Q

Organ failure associated with multiple organ dysfunction syndrome (MODS) usually begins in which organ?

A

Lungs
Explanation:
During MODS, the organ failure usually begins in the lungs and is followed by failure of the liver, gastrointestinal system, and kidneys.

157
Q

Which type of burn injury requires skin grafting?

A

Full-thickness
Explanation:
A full-thickness burn injury heals by contraction or epithelial migration and requires grafting. The other types of burn injury do not require skin grafting.

158
Q

The client is admitted with full-thickness burns to the forearm. Which is the most accurate interpretation made by the nurse?

A

Skin grafting will be necessary.
Explanation:
In a full-thickness burn, all layers of the skin are destroyed and will result in the need for skin grafts. Full-thickness burns are painless. A deep partial-thickness burn may take 3 or more weeks to heal. In the most serious full-thickness burns, ligaments, tendons, muscles, and bone may be involved.

159
Q

Which of the following is the effect of protein catabolism in a client with severe burns?

A

It compromises wound healing and immunocompetence.
Explanation:
Protein catabolism in a client with severe burns compromises wound healing and immunocompetence. Burns of the face, neck, or chest have the potential to impair ventilation, while burns involving the hands or major joints may affect dexterity and mobility. Release of aldosterone, not protein catabolism, causes sodium retention.

160
Q

A client received burns to his entire back and left arm. Using the Rule of Nines, the nurse can calculate that he has sustained burns on what percentage of his body?

A

27%
Explanation:
According to the Rule of Nines, the posterior trunk, anterior trunk, and legs each make up 18% of the total body surface. The head, neck, and arms each make up 9% of total body surface, and the perineum makes up 1%. In this case, the client received burns to his back (18%) and one arm (9%), totaling 27% of his body.

161
Q

A client who has been burned significantly is taken by air ambulance to the burn unit. What physiologic process furthers a burn injury?

A

inflammatory
Explanation:
The initial burn injury is further extended by inflammatory processes that affect layers of tissue below the initial surface injury.

162
Q

A client is brought to the emergency department with partial-thickness and full-thickness burns on the left arm, left anterior leg, and anterior trunk. Using the Rule of Nines, what is the total body surface area that has been burned?

A

36%
Explanation:
The Rule of Nines divides body surface area into percentages that, when totaled, equal 100%. According to the Rule of Nines, the arms account for 9% each, the anterior legs account for 9% each, and the anterior trunk accounts for 18%. Therefore, this client’s burns cover 36% of the body surface area.

163
Q

A nurse practitioner administers first aid to a patient with a deep partial-thickness burn on his left foot. The nurse describes the skin involvement as the:

A

Epidermis and a portion of deeper dermis.
Explanation:
A deep partial-thickness burn includes the epidermis, upper dermis, and a portion of the deeper dermis. A burn limited to the epidermal layer is classified as a superficial partial-thickness burn. The last two choices refer to a full-thickness burn.

164
Q

When using the Palmer method to estimate the extent of the burn injury, the nurse determines the palm is equal to which percentage of total body surface area?

A

1
Explanation:
In clients with scattered burns, or for a quick prehospital assessment, the Palmer method may be used to estimate the extent of the burns. The size of the client’s palm, including the surface area of the digits, is approximately 1% of the total body surface area.

165
Q

The nurse participates in a health fair about fire safety. When clothes catch fire, which intervention helps to minimize the risk of further injury to an affected person at a scene of a fire?

A

Roll the client in a blanket.
Explanation:
When clothing catches fire, the flames can be extinguished if the person drops to the floor or ground and rolls (“stop, drop, and roll”); anything available to smother the flames, such as a blanket, rug, or coat, may be used. The older adult, or others with impaired mobility, could be instructed to “stop, sit, and pat” to prevent concomitant musculoskeletal injuries. The client should not be covered immediately with a wet cloth or kept in any position other than horizontal. However, IV fluid therapy should be administered en route to the hospital.

166
Q

Which of the following neuroendocrine changes occur within the first 24 hours of a serious burn?

A

Hyperglycemia
Explanation:
When the adrenal cortex is stimulated, it releases glucocorticoids , which cause hyperglycemia. Sodium retention leads to peripheral edema. There is a decreased urine output, initially.

167
Q

Which type of debridement occurs when nonliving tissue sloughs away from uninjured tissues?

A

Natural
Explanation:
Natural debridement is accomplished when nonliving tissue sloughs away from uninjured tissue. Mechanical debridement involves the use of surgical tools to separate and remove the eschar. Enzymatic debridement encompasses the use of topical enzymes to the burn wound. Surgical debridement uses the use of forceps and scissors during dressing changes or wound cleaning.

168
Q

A client with a severe electrical burn injury is treated in the burn unit. Which laboratory result would cause the nurse the most concern?

A

BUN: 28 mg/dL
Explanation:
The elevated BUN would cause the nurse the most concern. The nurse should report decreased urine output or increased BUN and creatinine values to the physician. These laboratory values indicate possible renal failure. In addition, myoglobinuria, associated with electrical burns, is common with muscle damage and may also cause kidney failure if not treated. The other values are within normal limits.

169
Q

As the first priority of care, a patient with a burn injury will initially need:

A

a patent airway established.
Explanation:
Breathing must be assessed and a patent airway established immediately during the initial minutes of emergency care. Immediate therapy is directed toward establishing an airway and administering humidified 100% oxygen.

170
Q

A client has partial-thickness burns on both lower extremities and portions of the trunk. Which IV fluid does the nurse plan to administer first?

A

Lactated Ringer’s solution
Explanation:
Lactated Ringer’s solution replaces lost sodium and corrects metabolic acidosis, both of which commonly occur following a burn. Albumin is used as adjunct therapy, not as primary fluid replacement. D5W isn’t given to burn clients during the first 24 hours because it can cause pseudodiabetes. The client is hyperkalemic as a result of the potassium shift from the intracellular space to the plasma, so giving potassium would be detrimental.

171
Q

The nurse knows that inflammatory response following a burn is proportional to the extent of injury. Which factor presents the greatest impact on the ability to modify the magnitude and duration of the inflammatory response in a client with a burn?

A

Preexisting conditions
Explanation:
Preexisting disease disorders including trauma and infections can modify the inflammatory response and movement of fluid from the vascular to the interstitial space. Age, weight, and family history are not as significant in the inflammatory response following a burn.

172
Q

When assessing a client with partial-thickness burns over 60% of the body, which finding should the nurse report immediately?

A

Hoarseness of the voice
Explanation:
Hoarseness is indicative of injury to the respiratory system and could indicate the need for immediate intubation. Thirst following burns is expected because of the massive fluid shifts and resultant loss, leading to dehydration. Pain, either severe or moderate, is expected with a burn injury. The client’s urine output is adequate.

173
Q

In a client who has been burned, which medication should the nurse expect to use to prevent infection?

A

Mafenide (Sulfamylon)
Explanation:
The topical antibiotic mafenide is ordered to prevent infection in clients with partial-thickness and full-thickness burns. Lindane is a pediculicide used to treat lice infestation. Diazepam is an antianxiety agent that may be administered to clients with burns, but not to prevent infection. The opioid analgesic meperidine is used to help control pain in clients with burns.

174
Q

A nurse is aware that after a burn injury and respiratory difficulties have been managed, the next most urgent need is to:

A

Replace lost fluids and electrolytes.
Explanation:
After managing respiratory difficulties, the next most urgent need is to prevent irreversible shock by replacing lost fluids and electrolytes. The total volume and rate of IV fluid replacement are gauged by the patient’s response and guided by the resuscitation formula.

175
Q

A nurse is required to monitor the effectiveness of fluid resuscitation in a client who is being treated for burns. Which of the following assessments would indicate the success of the fluid resuscitation?

A

The client’s urinary output is 0.5 mL/kg/hour.
Explanation:
Successful fluid resuscitation is gauged by a urinary output of 0.5 mL/kg/hour via an indwelling catheter. Fluid resuscitation does not directly affect the client’s heart rate, breathing, or mental status.

176
Q

A client is brought to the ED with burns exceeding 20% of total body surface area. Which is the primary nursing intervention in the care of this client

A

Fluid resuscitation
Explanation:
Fluid resuscitation requirements are paramount in the management of clients having burns that exceed 20% of TBSA. Fluid resuscitation with crystalloid and colloid solutions is calculated from the time the burn injury occurred to restore the intravascular volume and prevent hypovolemic shock and renal failure. Infection prevention is a care consideration with all burns. Endotracheal tube placement may be necessary if respiratory factors indicate the need. Intake and output records are maintained to determine the success of fluid resuscitation efforts.

177
Q

The nurse has completed teaching home care instructions to a client being discharged from the burn unit. Which statement from the client indicates the need for further teaching?

A

“As my wound heals, my skin will be itchy; I can apply lotion if scratching doesn’t help.”
Explanation:
Itching is a normal part of healing. Many clients describe this as one of the most uncomfortable aspects of burn recovery. The client can apply mild moisturizers to decrease itching from dryness. Medications can be discussed with your treatment team. The client should pat the areas; scratching is contraindicated. The other statements indicate that teaching has been effective.

178
Q

A nurse knows to assess a patient with a burn injury for gastrointestinal complications. Which of the following is a sign that indicates the presence of a paralytic ileus?

A

Decreased peristalsis
Explanation:
Decreased peristalsis and hypoactive bowel sounds are manifestations of a paralytic ileus.

179
Q

A client with a burn wound is prescribed mafenide acetate 5% twice daily. Nursing implications associated with this medication include

A

premedicating the client with an analgesic prior to application.
Explanation:
Mafenide is a strong carbonic anhydrase inhibitor and may cause metabolic acidosis. Application may cause considerable pain initially, thus premedicating the client is an appropriate intervention. The other nursing implications are not associated with mafenide.

180
Q

Which of the following measures can be used to cool a burn?

A

Application of cool water
Explanation:
Once a burn has been sustained, the application of cool water is the best first-aid measure. Never apply ice directly to the burn, never wrap the person in ice, and never use cold soaks or dressings for longer than several minutes; such procedures may worsen the tissue damage and lead to hypothermia in people with large burns.

181
Q

A client has a burn on the leg related to an engine fire. When the burn area was assessed, it was determined that the client felt no pain in the area and that it appeared leathery. How would the nurse document the depth of burn injury this client has?

A

full thickness (third degree)
Explanation:
Full-thickness (third degree) burn destroys all layers of the skin and consequently is painless. The tissue appearance varies and can be dry, pale white, red, brown, leathery, charred or lifeless. Superficial (first degree) burn is similar to a sunburn. The epidermis is injured, but the dermis is unaffected. Superficial partial-thickness burn heals within 14 days, with possibly some pigmentary changes but no scarring. The deep partial-thickness (second degree) burn takes more than 3 weeks to heal, may need debridement, and is subject to hypertrophic scarring. A fourth-degree burn can involve ligaments, tendons, muscles, nerves, and bone.

182
Q

Immediately after a burn injury, electrolytes need to be evaluated for a major indicator of massive cell destruction, which is:

A

Hyperkalemia.
Explanation:
Circulating blood volume decreases dramatically during burn shock due to severe capillary leak with variation of serum sodium levels in response to fluid resuscitation. Usually, hyponatremia (sodium depletion) is present. Immediately after burn injury, hyperkalemia (excessive potassium) results from massive cell destruction. Hypokalemia (potassium depletion) may occur later with fluid shifts and inadequate potassium replacement.

183
Q

Following a burn injury, the nurse determines which area is the priority for nursing assessment?

A

Pulmonary system
Explanation:
Airway patency and breathing must be assessed during the initial minutes of emergency care. Immediate therapy is directed toward establishing an airway and administering humidified 100% oxygen. Pulmonary problems may be caused by the inhalation of heat and/or smoke or edema of the airway. Assessing a patent airway is always a priority after a burn injury followed by breathing. Remember the ABCs.

184
Q

A client who has sustained burns to the anterior chest and upper extremities is brought to the burn center. During the initial stage of assessment, which nursing diagnosis is primary?

A

Risk for Impaired Gas Exchange
Explanation:
During the initial assessment of a burn victim, the nurse must look for evidence of inhalation injury. Once oxygen saturation and respirations are determined, pain intensity is evaluated. The assessment of damage to the tissues and prevention of infection are secondary to airway issues.

185
Q

Which of the following is to be expected soon after a major burn? Select all that apply.

A

Tachycardia, slight hypotension, and anxiety are expected soon after the burn.

186
Q

The most important intervention in the nutritional support of a client with a burn injury is to provide adequate nutrition and calories. The nurse recognizes this intervention is to promote

A

decreased catabolism.
Explanation:
Burn injuries produce profound metabolic abnormalities fueled by the exaggerated stress response to the injury. The body’s response has been classified as hyperdynamic, hypermetabolic, and hypercatabolic. The most important intervention in the nutritional support of a client with a burn injury is to provide adequate nutrition and calories to decrease catabolism. Nutritional support with optimized protein intake can decrease the protein losses by approximately 50%. A marked increase in metabolic rate is seen after a burn injury and interventions are instituted to decrease metabolic rate and catabolism. A marked increase in glucose demand is seen after a burn injury and interventions are instituted to decrease glucose demands and catabolism. Rapid skeletal muscle breakdown with amino acids serving as the energy source is seen after a burn injury and interventions are instituted to decrease catabolism.

187
Q

A person suffers leg burns from spilled charcoal lighter fluid. A family member extinguishes the flames. While waiting for an ambulance, what should the burned person do?

A

Have someone assist him into a bath of cool water, where he can soak intermittently while waiting for emergency personnel.
Explanation:
After the flames are extinguished, the burned area and adherent clothing are soaked with cool water, briefly, to cool the wound and halt the burning process.

188
Q

The nurse is caring for a client who has sustained severe burns to 50% of the body. The nurse is aware that fluid shifts during the first week of the acute phase of a burn injury cause massive cell destruction. What should the nurse report if it occurs immediately after burn injury?

A

Hyperkalemia
Explanation:
Immediately after burn injury, hyperkalemia (excessive potassium) may result from massive cell destruction. Hypokalemia (potassium depletion) may occur later with fluid shifts and inadequate potassium replacement. During burn shock, serum sodium levels vary in response to fluid resuscitation. Hyponatremia (serum sodium depletion) may be present as a result of plasma loss. Hyponatremia may also occur during the first week of the acute phase, as water shifts from the interstitial space and returns to the vascular space.

189
Q

Which is the primary reason for placing a client in a horizontal position while smothering flames are present?

A

To keep fire and smoke from airway
Explanation:
The primary reason the client is placed in a horizontal position while smothering flames is to prevent the fire, hot air, and smoke from rising toward the head and entering the respiratory passages. The stop, drop, and roll method is a quick and efficient means to extinguish flames. If hypovolemic shock occurs, lowering the head will assist in promoting blood flow to the head.